Finite subset of a metric space

Click For Summary
In the discussion about limit points of a finite subset E of an infinite set X with a defined metric, it is concluded that there are no limit points for E. The reasoning is that for any point p, there exists a neighborhood B(p, 1/n) that does not contain any other points of E, satisfying the definition of a limit point. Participants clarify that while B(p, 1/n) contains p, it does not necessarily include other points from E for some n. Additionally, there is a request for proofs regarding finite sets being closed and compact in metric spaces, which prompts a suggestion to create a new topic for that discussion. The conversation emphasizes the importance of careful consideration of neighborhood definitions in metric spaces.
rjw5002

Homework Statement



Let X be an infinite set. For p,q \in X define:
d(p,q) = {1 if p \neq q; 0 if p = q
Suppose E is a finite subset of X, find all limit points of E.

Homework Equations



definition: a point p is a limit point of E if every neighborhood of p contains a point q \neq p s.t. q\inE

The Attempt at a Solution


My thoughts were that there are no limit points for any subset of X because for any point p, B(p, 1/n) is empty, \foralln\inN. Therefore, for any point p, there exists at least one neighborhood that contains no points q \in E.

I feel that this is true, but have I made any incorrect assumptions??
Thanks a lot for any feedback.
 
Physics news on Phys.org
B(p,1/n) isn't empty for all n. For instance p is always in B(p,1/n), and if n=1, then B(p,1/n) is in fact all of X.

You definitely have the right idea, but you have to be a bit more careful.
 
remember that you have the additional rquirement that q\ne p. I think that is what you meant to say. For all n, B(p, 1/n) contains p, but for some n, B(p, 1/n) does not include any other point of the set.
 
Right, ok. I can fix that detail. Thanks a lot guys.
 
i need the proof for every finite set is a closed set in a metric space ..
and finite set in a metric space in compact as soon as possible../.
 
nehakapoor said:
i need the proof for every finite set is a closed set in a metric space ..
and finite set in a metric space in compact as soon as possible../.

You will need to make a new topic for this. And it would also be nice to see your attempt...
 
Question: A clock's minute hand has length 4 and its hour hand has length 3. What is the distance between the tips at the moment when it is increasing most rapidly?(Putnam Exam Question) Answer: Making assumption that both the hands moves at constant angular velocities, the answer is ## \sqrt{7} .## But don't you think this assumption is somewhat doubtful and wrong?

Similar threads

  • · Replies 24 ·
Replies
24
Views
4K
  • · Replies 4 ·
Replies
4
Views
2K
  • · Replies 4 ·
Replies
4
Views
2K
  • · Replies 7 ·
Replies
7
Views
2K
  • · Replies 4 ·
Replies
4
Views
2K
  • · Replies 5 ·
Replies
5
Views
2K
  • · Replies 4 ·
Replies
4
Views
3K
  • · Replies 2 ·
Replies
2
Views
4K
Replies
3
Views
2K
  • · Replies 11 ·
Replies
11
Views
3K